Lösung.

Sei $ \mbox{$S\in\mathbb{C}^{n\times n}$}$ invertierbar mit $ \mbox{$S^{-1}AS=J$}$ in Jordanform.

1.
Sei zunächst $ \mbox{$A^m=0$}$ . Es ist $ \mbox{$J^m=(S^{-1}AS)^m=S^{-1}A^mS=0$}$ und folglich $ \mbox{$(\text{J}_k(\lambda))^m=0$}$ für jeden auftretenden Jordanblock $ \mbox{$\text{J}_k(\lambda)$}$ . Die Hauptdiagonaleinträge von $ \mbox{$(\text{J}_k(\lambda))^m$}$ sind alle gleich $ \mbox{$\lambda^m$}$ . Aus $ \mbox{$\lambda^m=0$}$ folgt nun $ \mbox{$\lambda=0$}$ . Da dies für jeden auftretenden Jordanblock gilt, ist $ \mbox{$0$}$ der einzige Eigenwert.

Sei nun umgekehrt $ \mbox{$0$}$ einziger Eigenwert von $ \mbox{$A$}$ . Dann sind alle auftretenden Jordanblöcke der Form $ \mbox{$\text{J}_k(0)$}$ für gewisse $ \mbox{$k\geq 1$}$ , und es ist $ \mbox{$(\text{J}_k(0))^k=0$}$ . Sei $ \mbox{$m$}$ die maximale Kantenlänge aller auftretenden Jordanblöcke. Dann ist $ \mbox{$J^m=0$}$ , und es folgt $ \mbox{$A^m=SJ^mS^{-1}=0$}$ .

Alternativ ist $ \mbox{$A^m=0$}$ für ein $ \mbox{$m\geq 1$}$ genau dann, wenn $ \mbox{$\mu_A(X)=X^l$}$ für ein $ \mbox{$l\geq 1$}$ , was wiederum genau dann gilt, wenn $ \mbox{$0$}$ einziger Eigenwert von $ \mbox{$A$}$ ist.

2.
Die Matrix $ \mbox{$A$}$ ist diagonalisierbar genau dann, wenn $ \mbox{$J$}$ eine Diagonalmatrix ist, was genau dann der Fall ist, wenn alle Jordanblöcke die Kantenlänge eins haben, was schließlich genau dann der Fall ist, wenn $ \mbox{$\mu_A(X)$}$ nur einfache Nullstellen hat.

3.
Sei $ \mbox{$\chi_A(X)=(X-\lambda_1)^{m_1}\cdots(X-\lambda_r)^{m_r}$}$ und $ \mbox{$\mu_A(X)=(X-\lambda_1)^{l_1}\cdots(X-\lambda_r)^{l_r}$}$ , wobei $ \mbox{$\lambda_1,\dots,\lambda_r$}$ paarweise verschieden seien. Es ist die Summe der Kantenlängen aller Jordanblöcke zum Eigenwert $ \mbox{$\lambda_i$}$ gleich $ \mbox{$m_i$}$ .

Daher ist $ \mbox{$\chi_A(X)=\mu_A(X)$}$ genau dann, wenn $ \mbox{$m_i=l_i$}$ ist für alle $ \mbox{$i$}$ , was genau dann der Fall ist, wenn für jedes $ \mbox{$i$}$ genau ein Jordanblock existiert, und zwar mit Kantenlänge $ \mbox{$m_i$}$ , was schließlich dazu äquivalent ist, daß jeder Eigenwert geometrische Vielfachheit eins besitzt.

4.
Aus $ \mbox{$A^m = \text{E}$}$ folgt $ \mbox{$J^m = S^{-1} A^m S = \text{E}$}$ . Träte in $ \mbox{$J$}$ ein Jordanblock $ \mbox{$\text{J}_k(\lambda)$}$ mit $ \mbox{$k\geq 2$}$ auf, so wäre $ \mbox{$(\text{J}_k(\lambda))^m = \text{E}$}$ . Auf der ersten oberen Nebendiagonalen von $ \mbox{$(\text{J}_k(\lambda))^m$}$ findet sich der Eintrag $ \mbox{$m\lambda^{k-1}$}$ , wie eine Induktion zeigt. Aus $ \mbox{$m\lambda^{k-1} = 0$}$ folgt nun $ \mbox{$\lambda=0$}$ . Nun steht aber in der Hauptdiagonalen von $ \mbox{$(\text{J}_k(\lambda))^m$}$ eine Null, im Widerspruch zu $ \mbox{$(\text{J}_k(\lambda))^m = \text{E}$}$ . Also haben alle Jordanblöcke Kantenlänge eins, und es folgt $ \mbox{$A$}$ diagonalisierbar.

Alternativ, mit $ \mbox{$f(X):=X^m-1$}$ gilt $ \mbox{$f(A)=0$}$ . Daher ist $ \mbox{$\mu_A(X)$}$ ein Teiler von $ \mbox{$X^m-1$}$ . Die Nullstellen von $ \mbox{$f(X)$}$ sind genau die $ \mbox{$m$}$ -ten Einheitswurzeln $ \mbox{$\zeta_m^k:=\exp(2\pi\mathrm{i}k/m)$}$ für $ \mbox{$k\in\{0,\dots,m-1\}$}$ , und diese sind paarweise verschieden. Es folgt $ \mbox{$X^m-1=(X-\zeta_m^0)\cdots(X-\zeta_m^{m-1})$}$ . Also besitzt $ \mbox{$f(X)$}$ und somit auch $ \mbox{$\mu_A(X)$}$ nur einfache Nullstellen, und folglich ist $ \mbox{$A$}$ diagonalisierbar.

5.
Mit $ \mbox{$f(X):=X^2-2X+1=(X-1)^2$}$ ist $ \mbox{$f(A)=0$}$ . Somit ist $ \mbox{$\mu_A(X)=X-1$}$ oder $ \mbox{$\mu_A(X)=(X-1)^2$}$ . Im Falle $ \mbox{$\mu_A(X)=X-1$}$ ist wegen $ \mbox{$\mu_A(A)=0$}$ mithin $ \mbox{$A=\text{E}$}$ . Im Falle $ \mbox{$\mu_A(X)=(X-1)^2$}$ hat $ \mbox{$\mu_A(X)$}$ eine doppelte Nullstelle, und daher ist $ \mbox{$A$}$ dann nicht diagonalisierbar.

Zum Beispiel erfüllt die Matrix $ \mbox{$A:=\begin{pmatrix}1&1\\  0&1\end{pmatrix}\ne\text{E}$}$ die Gleichung $ \mbox{$A^2=2A-\text{E}$}$ .